Quesito 8 – Testo e soluzione – Maturità 2010 scientifico PNI

Testo

Se n>3 e \[ a_{n-1}=\left(\begin{array}{c} n\\ n-1 \end{array}\right),a_{n-2}=\left(\begin{array}{c} n\\ n-2 \end{array}\right),a_{n-3}=\left(\begin{array}{c} n\\ n-3 \end{array}\right) \] sono in progressione aritmetica, che valore avrà n?

Soluzione

Se i tre elementi sono in progressione aritmetica significa che la differenza tra un elemento della successione e il precedente – o il successivo – è un valore costante d che si chiama ‘ragione’. \[ a_{n-1}-a_{n-2}=a_{n-2}-a_{n-3}=d \] Svolgendo i coefficienti binomiali e ricordando che x>3 avrò: \[ n-\frac{n\left(n-1\right)}{2}=\frac{n\left(n-1\right)}{2}-\frac{n\left(n-1\right)\left(n-2\right)}{6} \] \[ 1-\frac{\left(n-1\right)}{2}=\frac{\left(n-1\right)}{2}-\frac{n^{2}-3n+2}{6} \] \[ n^{2}-9n+14=0\rightarrow n_{1}=7\; n_{2}=2 \] Delle due soluzioni trovate solamente n = 7 rispetta la condizioni che ci ha dato il testo (n>3) ed è pertanto l’unico valore possibile di n affinchè si abbia una progressione aritmetica.

 

Lascia un commento

Il tuo indirizzo email non sarà pubblicato. I campi obbligatori sono contrassegnati *